Difference between revisions of "2019 AMC 12B Problems/Problem 13"

(Solution 2 (variant))
(Solution 2 (variant))
Line 15: Line 15:
 
<cmath>\sum_{i=1}^{\infty} \frac{1}{4^i} = \boxed{\textbf{(C) } \frac{1}{3}}</cmath>
 
<cmath>\sum_{i=1}^{\infty} \frac{1}{4^i} = \boxed{\textbf{(C) } \frac{1}{3}}</cmath>
  
-scrabbler94
+
(Note: to find this sum, we use the formula <math>\sum_{i=1}^{\infty} r^i = \frac{r}{1-r}</math>. Since in this case, <math>r = \frac{1}{4}</math>, the answer is <math>\frac{\frac{1}{4}}{\frac{3}{4}} = \frac{1}{3}</math>. If you don't know this formula, you may instead note that if you multiply the sum by <math>4</math>, it is equivalent to adding <math>1</math>. Thus: <math>4n = n+1</math>, which clearly simplifies to <math>n = \frac{1}{3}</math>.
 +
 
 +
- scrabbler94 (explanation of infinite sum provided by Robin)
  
 
==See Also==
 
==See Also==

Revision as of 16:15, 14 February 2019

The following problem is from both the 2019 AMC 10B #17 and 2019 AMC 12B #13, so both problems redirect to this page.

Problem

A red ball and a green ball are randomly and independently tossed into bins numbered with the positive integers so that for each ball, the probability that it is tossed into bin $k$ is $2^{-k}$ for $k = 1,2,3....$ What is the probability that the red ball is tossed into a higher-numbered bin than the green ball?
$\textbf{(A) } \frac{1}{4} \qquad\textbf{(B) } \frac{2}{7} \qquad\textbf{(C) } \frac{1}{3} \qquad\textbf{(D) } \frac{3}{8} \qquad\textbf{(E) } \frac{3}{7}$

Solution

The probability of the red ball landing in a higher-numbered bin is the same as the probability of the green ball landing in a higher numbered bin. The probability of both landing in the same bin is $\sum_{k=1}^{\infty}2^{-2k}$. The sum is equal to $\frac{1}{3}$. Therefore the other two probabilities have to both be $\textbf{(C) } \frac{1}{3}$.
$Q.E.D\blacksquare$
Solution by a1b2

Solution 2 (variant)

Suppose the green ball goes in bin $i$, for some $i \ge 1$. The probability of this occurring is $\frac{1}{2^i}$. Given this occurs, the probability that the red ball goes in a higher-numbered bin is $\frac{1}{2^{i+1}} + \frac{1}{2^{i+2}} + \ldots = \frac{1}{2^i}$. Thus the probability that the green ball goes in bin $i$, and the red ball goes in a bin greater than $i$, is $\left(\frac{1}{2^i}\right)^2 = \frac{1}{4^i}$. Summing from $i=1$ to infinity, we get

\[\sum_{i=1}^{\infty} \frac{1}{4^i} = \boxed{\textbf{(C) } \frac{1}{3}}\]

(Note: to find this sum, we use the formula $\sum_{i=1}^{\infty} r^i = \frac{r}{1-r}$. Since in this case, $r = \frac{1}{4}$, the answer is $\frac{\frac{1}{4}}{\frac{3}{4}} = \frac{1}{3}$. If you don't know this formula, you may instead note that if you multiply the sum by $4$, it is equivalent to adding $1$. Thus: $4n = n+1$, which clearly simplifies to $n = \frac{1}{3}$.

- scrabbler94 (explanation of infinite sum provided by Robin)

See Also

2019 AMC 10B (ProblemsAnswer KeyResources)
Preceded by
Problem 16
Followed by
Problem 18
1 2 3 4 5 6 7 8 9 10 11 12 13 14 15 16 17 18 19 20 21 22 23 24 25
All AMC 10 Problems and Solutions
2019 AMC 12B (ProblemsAnswer KeyResources)
Preceded by
Problem 12
Followed by
Problem 14
1 2 3 4 5 6 7 8 9 10 11 12 13 14 15 16 17 18 19 20 21 22 23 24 25
All AMC 12 Problems and Solutions

The problems on this page are copyrighted by the Mathematical Association of America's American Mathematics Competitions. AMC logo.png